Ashton

This topic has expert replies
User avatar
Master | Next Rank: 500 Posts
Posts: 345
Joined: Sun Dec 21, 2008 10:57 pm
Thanked: 6 times
Followed by:3 members

Ashton

by himu » Mon Nov 05, 2012 9:14 pm
Despite the best efforts of our national soccer league, the city of Ashton refuses to adequately support its professional franchise, which has the lowest ratings, revenue, and season ticket sales of any in the league. Therefore it is clear that Ashton is not interested in soccer.

Which of the following, if true, would most weaken the argument?

Ashton's franchise has been in existence for over twenty years.

Ashton is nationally renowned for its professional basketball team, perhaps the most successful in the history of the country.

Ashton has two long established collegiate soccer programs that dominate media coverage in the city.

Ashton has the lowest average income per capita of any city in the league.

Ashton has refused to build a new stadium to support the franchise.

User avatar
Master | Next Rank: 500 Posts
Posts: 279
Joined: Mon Jun 25, 2012 10:56 pm
Thanked: 60 times
Followed by:10 members

by anuprajan5 » Mon Nov 05, 2012 9:56 pm
Answer C
Regards
Anup

The only lines that matter - are the ones you make!

https://www.youtube.com/watch?v=kk4sZcG ... ata_player

User avatar
Master | Next Rank: 500 Posts
Posts: 121
Joined: Fri Mar 13, 2009 8:44 am
Thanked: 9 times
Followed by:1 members
GMAT Score:610

by singh181 » Tue Nov 06, 2012 12:05 am
IMO D
D offers an alternate explanation.

Senior | Next Rank: 100 Posts
Posts: 58
Joined: Mon Aug 27, 2012 3:09 am
Thanked: 2 times

by akashkumar1987 » Tue Nov 06, 2012 2:33 am
Either B or C .....

User avatar
Master | Next Rank: 500 Posts
Posts: 111
Joined: Tue Jan 31, 2012 1:06 pm
Thanked: 15 times
Followed by:8 members

by charu_mahajan » Tue Nov 06, 2012 4:31 pm
IMO D
What is the OA?

Senior | Next Rank: 100 Posts
Posts: 38
Joined: Mon Mar 15, 2010 8:38 pm
Location: Mumbai
Thanked: 7 times
GMAT Score:710

by eki » Thu Nov 08, 2012 4:53 am
IMO its C.

conclusion : Ashton is not interested in soccer.

C shows that it has two long established collegiate soccer programs, which negates the above conlusion.
D is partially correct : low income means less ticket sales and revenue. But it can not account for low ratings.

Senior | Next Rank: 100 Posts
Posts: 38
Joined: Mon Mar 15, 2010 8:38 pm
Location: Mumbai
Thanked: 7 times
GMAT Score:710

by eki » Thu Nov 08, 2012 4:54 am
IMO its C.

conclusion : Ashton is not interested in soccer.

C shows that it has two long established collegiate soccer programs, which negates the above conlusion.
D is partially correct : low income means less ticket sales and revenue. But it can not account for low ratings.

Newbie | Next Rank: 10 Posts
Posts: 4
Joined: Fri Nov 02, 2012 6:10 pm

by chiranjeev12 » Thu Nov 08, 2012 8:23 am
himu wrote:Despite the best efforts of our national soccer league, the city of Ashton refuses to adequately support its professional franchise, which has the lowest ratings, revenue, and season ticket sales of any in the league. Therefore it is clear that Ashton is not interested in soccer.

Which of the following, if true, would most weaken the argument?

Ashton's franchise has been in existence for over twenty years.

Ashton is nationally renowned for its professional basketball team, perhaps the most successful in the history of the country.

Ashton has two long established collegiate soccer programs that dominate media coverage in the city.

Ashton has the lowest average income per capita of any city in the league.

Ashton has refused to build a new stadium to support the franchise.
The argument in the passage is:
Conclusion: Ashton is not interested in soccer
Premise: Despite the best efforts of our national soccer league, the city of Ashton refuses to adequately support its professional franchise.

So, the argument is like:
Since the city of Ashton refuses to adequately support its professional franchise, it is not interested in soccer.

Reading the above statement, it becomes clear that any statement which suggests that there could be other reasons for Ashton to not support its franchise than its lack of interest, would greatly weaken the argument.

Looking at the options, the answer is clearly option D which provides an alternate reasoning for Ashton not supporting its franchise. Since Ashton has the lowest average per capita income in the league, it may not be able to support its league due to financial constraints, rather than its lack of interest.

Some of us have mentioned option C, which is:
Ashton has two long established collegiate soccer programs that dominate media coverage in the city.
A weakener is the one which weakens the logic in the argument, not the one which falsifies the statements given. Option C says that Ashton does indeed support its franchise, which is against the information given in the passage. Thus, C cannot be the answer.

Cheers,
CJ

Master | Next Rank: 500 Posts
Posts: 423
Joined: Fri Jun 11, 2010 7:59 am
Location: Seattle, WA
Thanked: 86 times
Followed by:2 members

by srcc25anu » Fri Mar 08, 2013 6:43 am
+ 1 for C
Please post OA after the question.

Senior | Next Rank: 100 Posts
Posts: 55
Joined: Wed Oct 07, 2009 3:55 pm

by apoorva.rattan » Fri Mar 08, 2013 3:49 pm
IMO .. C

User avatar
Master | Next Rank: 500 Posts
Posts: 345
Joined: Sun Dec 21, 2008 10:57 pm
Thanked: 6 times
Followed by:3 members

by himu » Thu Mar 14, 2013 7:16 am
Solution: C
[spoiler]
Begin this Weaken question by identifying the conclusion, negating it, and looking for support for the negation. The negation here is "Ashton may be interested in soccer", and (C) provides evidence to support that assertion: just because Ashton doesn't care about PROFESSIONAL soccer doesn't mean that it doesn't care about any kind of soccer.[/spoiler]

Master | Next Rank: 500 Posts
Posts: 423
Joined: Fri Jun 11, 2010 7:59 am
Location: Seattle, WA
Thanked: 86 times
Followed by:2 members

by srcc25anu » Thu Mar 14, 2013 7:39 am
I don't have the OA for this question. I came across this question on btg forum which was posted mid of 2012 but had no OA. I also tried searching on google but could not find one. But I truly everybody who responded / attempted. The consensus is for choice C. Only if some expert could also confirm that would be great.